Können wir daraus schließen, dass die eine Sequenz $a_n$ so dass $ |a_1| \lt |a_2 -a_1| \lt |a_3 -a_2| \lt |a_4 - a_3| \dots$, und $a_1 \neq 0$ nimmt zu?

Aug 15 2020

Wir haben eine unendliche Folge $$ a_1, a_2 , a_3 \cdots $$ Und das ist gegeben $$ |a_1| \lt |a_2 -a_1| \lt |a_3 -a_2| \lt |a_4 - a_3| \cdots \\ a_1 \neq 0 $$ (das heißt, die Differenz zwischen den nachfolgenden Begriffen nimmt zu und der erste Begriff kann nicht Null sein)

Können wir daraus schließen, dass die absoluten Werte nachfolgender Terme zunehmen? Daraus können wir schließen$$ |a_1| \lt |a_2| \lt |a_3| \lt |a_4| \cdots $$ Das Herumspielen mit den in der Frage angegebenen Ungleichungen kann uns die Information geben, dass die alternativen Begriffe zunehmen (in absoluten / numerischen Werten, verlassen $a_1$ abgesehen davon, nicht zu vergleichen $a_1$mit irgendwelchen Begriffen, aber nur darauf achten, dass es nicht Null ist), aber nicht die aufeinander folgenden Begriffe. Ich denke also, wir können nicht schlussfolgern, dass die aufeinanderfolgenden Begriffe numerisch zunehmen.

Eine erklärende Antwort wird gesucht.

Antworten

2 Chrystomath Aug 15 2020 at 13:47

Betrachten Sie die Reihenfolge $b_n:=c_n(1-\tfrac{1}{n})$ wo $c_n\in\{+1,-1\}$und die Reihenfolge $a_n:=\sum_{i=1}^nb_i$.

Dann $|a_{n+1}-a_n|=|b_{n+1}|=1-\tfrac{1}{n+1}$ nimmt jedoch aufgrund der zufälligen Auswahl von zu $c_i$ Es ist nicht abzusehen, ob $a_n$nimmt zu oder ab. Hier ist ein Beispiel, das durch eine zufällige Auswahl von generiert wurde$c_n$.

4 BrianM.Scott Aug 15 2020 at 13:43

Ein Gegenbeispiel reicht aus, und Sie können eines mit nur drei Begriffen erstellen. Wenn Sie ein wenig weiter gehen und zeigen möchten, dass es nicht einmal einen Punkt geben muss, ab dem die Begriffe an absoluten Werten zunehmen, müssen Sie etwas härter arbeiten, aber nicht viel. Zum Beispiel lassen$a_1=1$und im Allgemeinen lassen

$$a_{n+1}=\begin{cases} a_n-n,&\text{if }n\text{ is odd}\\ a_n+n,&\text{if }n\text{ is even,} \end{cases}$$

damit Sie die Sequenz erhalten $1,0,2,-1,3,\ldots\;$;; es ist nicht schwer, dies durch Induktion zu zeigen$a_{2n-1}=n$ und $a_{2n}=1-n$ für alle $n\in\Bbb Z^+$. Offensichtlich$|a_{n+1}-a_n|=n$ zum $n\in\Bbb Z^+$, aber $|a_{2n}|=n-1<n=|a_{2n-1}|$ zum $n\in\Bbb Z^+$.